AAMC CBT7 and 7R OFFICIAL Q&A

This forum made possible through the generous support of SDN members, donors, and sponsors. Thank you.

Vihsadas

No summer
Moderator Emeritus
Lifetime Donor
10+ Year Member
5+ Year Member
15+ Year Member
Joined
Oct 17, 2007
Messages
5,474
Reaction score
56
This is the official Q&A thread for AAMC CBT7 and 7R.

Please post ONLY questions pertaining to AAMC CBT7 and 7R.
Out of respect for people who may not have completed the other exams, do not post questions or material from any other AAMC exam.

Please see this thread for the rules of order before you post.

Good luck on your MCAT!

Members don't see this ad.
 
can some one explain 134? i chose A but how come choice B can't be an answer too?
 
Members don't see this ad :)
Hi guys,
Another Bio question:

Regarding Passage 7 (Boy I thought it was hard lol)

For item 136, why can't B be a viable answer? The fate of isolated P cells does equal the fate of normal P cells, no? That's even what it says in the answer solution for A.
did you ever figure this out? i was torn between choosing A or B
 
On #115, about the skin blood flow changing because of a change in the sympathetic nerves: I chose A through process of elimination, which is the correct answer, but even A confused me. When a nerve releases a neurotransmitter to its target tissue (in this case, norepinephrine to the smooth muscles of the blood vessel), then isn't the neurotransmitter either broken down in the synapse or reabsorbed by the pre-synaptic terminal? Does it go into the blood, as A suggests?
 
can some one explain 134? i chose A but how come choice B can't be an answer too?

I didn't choose (B) because I thought it was too extreme? Usually, bench research experiments like this are set up to disprove something, not prove. Think about it this way. Just because P1 gave rise to all of the same tissues in both the intact embryo and isolated situations doesn't mean that cultured and not cultured P1 cells are the same. It's much harder to prove that two things are identical (everything is the same!) than to prove that two things are different (there only needs to be one thing different!)

In short, choice (B) is too radical an answer given the simple data provided in the passage.

On #115, about the skin blood flow changing because of a change in the sympathetic nerves: I chose A through process of elimination, which is the correct answer, but even A confused me. When a nerve releases a neurotransmitter to its target tissue (in this case, norepinephrine to the smooth muscles of the blood vessel), then isn't the neurotransmitter either broken down in the synapse or reabsorbed by the pre-synaptic terminal? Does it go into the blood, as A suggests?

Not sure if it goes into the blood. But just like you said, the other answers don't make any sense...If anything, I only chose (A) because sympathetic activity is related to norepinephrine levels.
 
For the simple ox-red question #19 in physical sciences, I completely understand the answer and did the addition right, however I thought you had to multiple the electrode potential by the # of moles. So +0.34x2 moles Cu2+ - could someone clarify this for me? From the answer I see the multiplying isn't necessary but I swear in the berkeley books they have an example where you do..
 
AAMC 7R

14 – I know that the passage states that electrolytic techniques did not work well. But there is not further explanation about this technique in the passage. I didn’t know what this technique does – I was thinking about electrolytic cells and how it involves ions so I picked choice D – it does not crystallize well.

18 – SiCl3H when I draw this molecule out, it is a tetrahedral, Si in the centre with 3 Cl and 1 H atom attached so there is a net dipole moment which means that it is poplar – a type of covalent bond. I know that a low BP is mentioned in the Q stem but even covalent bonds have low BP (comparing to ionic bonds)

28 – Can someone go over how this calculation is to be done?

I did A = Ecl = (198)(4.14x10^-3)(1) = approx. 0.8 (choice B)

31 – when determining mass given the density, why are they assuming 1L as the volume?

I know how to do this calculation but I was confused because the volume was not provided in the passage.

39 – I didn’t know what “discharge” means. I thought that as current/charge flows from battery to capacitor, it stores it. So discharge means that current would be let go from the capacitor.

Can someone explain how capacitors work? I am confused.

40 – PE<w done by battery because some energy is lost due to heating of the resistor between the battery and the capacitor.

So, if there was no resistor, would PE = W done by battery?

Do capacitors also heat up in the process of accumulating charge?

65- can someone go over how to set up the formula for this question?
 
For the simple ox-red question #19 in physical sciences, I completely understand the answer and did the addition right, however I thought you had to multiple the electrode potential by the # of moles. So +0.34x2 moles Cu2+ - could someone clarify this for me? From the answer I see the multiplying isn't necessary but I swear in the berkeley books they have an example where you do..

Electrochemical potential is an intrinsic property and does not depend on the number of moles. This is different from enthalpy in Hess's Law where H needs to be multiplied by the number of moles used in the reaction.
 
  • Like
Reactions: 1 user
178 - skin blood flow - why is choice B wrong?

A - I know that change in norepinephrine (NE) in blood would reduce blood flow to the skin but ONLY if the change is in NE concentration is positive, leading to constriction of blood vessels.

B seemed to be a more inclusive answer
ACH = contraction of smooth muscle = decreased blood flow in blood vessels
 
Last edited:
174 - is this something we should know? I thought that the question was totally random.
 
Can someone explain Q208 (old 7R test) - I was confused about choices A and C - basically interpreting about chemist 1 and chemist 2 from the passage.
 
Members don't see this ad :)
This is what the reaction looked like:
2 Cu2+ + 2 H2O &#8594; 2 Cu(s) + O2 + 4 H+
The proper equation for determining Ecell is to ADD the potentials, not to subtract them (Ecell=Eanode+Ecathode). +0.34V represents copper being reduced while -1.23V represents water being oxidized; this is exactly what is occurring in the given formula for which we need to determine Ecell. Therefore, we only need to add the potentials: -1.23V + 0.34V = -0.89V = Ecell.


Does this help?
Just to be sure...

Does this mean when the reactions are presented in that manner, the value they give us has the sign reversed if need be?

Like for ex, Cu2+2e-->Cu is 0.34V
but if they gave it to me written as

Cu--> Cu2+ +2e they will write -0.34? Because that is the reason I go it wrong, I couldn't figure out if they gave it to me as a reduction potential or an oxidation potential.
 
damn...one point away from getting an 11 on that bio section!
woops...i messed up the math again. I DID get an 11!!!
by far the highest score on any section done so far.

so my scores for each part of the paper test for AAMC 7 were:
PS: 8
VR: 7
BS: 11

composite = 26
thats good enough for my DO schools =]
You know I got the same score, and it is irritating me because each question that I got wrong, I delved into WAY too much detail thinking about it, easily 5 questions in PS like that. 2 I changed it to the correct answer and for some odd reason it didn't save. Library Wi-fi sucks.

Also for VR that stupid passage about the flowers really got me.
 
Just to be sure...

Does this mean when the reactions are presented in that manner, the value they give us has the sign reversed if need be?

Like for ex, Cu2+2e-->Cu is 0.34V
but if they gave it to me written as

Cu--> Cu2+ +2e they will write -0.34? Because that is the reason I go it wrong, I couldn't figure out if they gave it to me as a reduction potential or an oxidation potential.

oh em gee. you quoted me from a post from 6 years ago so i got the alert but WOW i have NO IDEA anymore what i wrote up there. it looks like gibberish and certainly nothing i could have written but lo and behold it is in fact moi. can you see now how little any of this mattered through med school? i'm now a 2nd year internal medicine resident doing really well at a great program sooo yeah haha i wish i could explain further but i can't anymore!!
 
  • Like
Reactions: 1 user
oh em gee. you quoted me from a post from 6 years ago so i got the alert but WOW i have NO IDEA anymore what i wrote up there. it looks like gibberish and certainly nothing i could have written but lo and behold it is in fact moi. can you see now how little any of this mattered through med school? i'm now a 2nd year internal medicine resident doing really well at a great program sooo yeah haha i wish i could explain further but i can't anymore!!

*Applause*

That's some serious SDN dedication.
 
  • Like
Reactions: 1 user
oh em gee. you quoted me from a post from 6 years ago so i got the alert but WOW i have NO IDEA anymore what i wrote up there. it looks like gibberish and certainly nothing i could have written but lo and behold it is in fact moi. can you see now how little any of this mattered through med school? i'm now a 2nd year internal medicine resident doing really well at a great program sooo yeah haha i wish i could explain further but i can't anymore!!

Thanks for replying LovelyBRass, congratulations on being a resident and doing well! I just wanted to bring it up so if anyone could reply that would be awesome. But definitely, applause on replying and the dedication to SDN. :) All the best.
 
For #33, it doesn't actually matter that they're asking about the rate of formation of Cl-, because Cl- is a product and isn't present in the rate law. The only change in the reaction conditions is that change in the H+ concentration. Since [H+] has decreased by a factor of 10 and is second order, the rate itself will decrease by a factor of 100, giving us our answer.

In other words, even if they asked about the rate of formation of Br2 or H2O, we would've approached this question the same way - in effect, they're just saying "what does changing [H+] do to the rate?"
 
Just to expand a bit on what the above poster said, the reason we are considering [H+] concentration has decreased by a factor of ten is because pH went from 1 to 2. In other words [H+] went from 10^-1 to 10^-2 (a 10 times decrease). Since the rest of rate law is not affected by this pH change the only thing affecting the rate of formation of the product is [H+], which is second order therefore the rate decreases by a factor of [H+]^2 or [10]^2 (or 100). The original rate was 1 x 10^-2 so a 100 times decrease will lead to 1 x 10^-4.
 
I actually got this one right, but i had it narrowed down to both A and B... I originally had it at A, but changed it to B just based on the fact that I figured they were leaning more for us to get that from the passage...
sl7rjH
 
Verbal question here. I don't understand how #75 is B? The passage doesn't give much detail here, only "policies at all levels should encourage the coordination of employer-provided training and broader schooling.” B didn't seem to be really coordinating anything and when I read "at all levels" in the passage I thought of levels of government so chose A. Guess I am reading too much into it? Still don't see much support in the passage for repaying tuition though.
upload_2014-12-23_11-56-49.png
 
The problem with A is that the passage definitely never supports the idea of penalizing companies for failing to train workers. The quote you provided is a great example - the author favors the encouragement of training, which still fits best with choice B. It's very true that the idea of repaying tuition never appears in the passage, but it would still be a viable solution to the issue prevented in the question stem. (Basically, choice B directly allows the employer to train his workers himself, unlike choices A and C; choice D is too far out of scope.)

Good question, though. I've always thought that one was a bit ambiguous compared to most AAMC verbal questions.
 
  • Like
Reactions: 1 user
178 - skin blood flow - why is choice B wrong?

A - I know that change in norepinephrine (NE) in blood would reduce blood flow to the skin but ONLY if the change is in NE concentration is positive, leading to constriction of blood vessels.

B seemed to be a more inclusive answer
ACH = contraction of smooth muscle = decreased blood flow in blood vessels

I also thought this. I thought \that A (the correct answer) actually seemed wrong because I didn't understand why NE would be in the blood.

Anyone know why B is incorrect? Is it because AcH would only be released by preganglionic neurons, not by the postganglionic neurons onto the smooth muscle? That's all I could think of.
 
Last edited:
Anyone know why B is incorrect? Is it because AcH would only be released by preganglionic neurons, not by the postganglionic neurons onto the smooth muscle? That's all I could think of.
Yes, I think this is the key. #178 (116 in the newer version) does seem a little ambiguous. But if the muscle cell response was due to sympathetic neurons like the question stem says, then the muscle cells would be responding to norepinephrine not ACh.

I wasn't sure about choice A because I never heard of neurotransmitters leaking out from motor neuron junctions into the blood but apparently it can happen. If anyone can elaborate on this leaking out process I would be love to hear about it.
 
I have no idea what #46 (AAMC #7 CBT) is asking. Can anyone help me picture it? I don't understand what the question means by saying the "rupture" goes from "south to north". When I picture an earthquake, there is a central point where seismic waves all originate from and they propagate in all directions. I don't know if this is relevant to the question stem. If anyone can help me out, it would be much appreciated. Thanks.
 
@ieatshrimp24 the question states it is not a static epicenter but moving rupture. For example it could be like a fracture along a fault line that starts at one point and moves north:
earthquake-plate-tectonic-iceland-story.jpg

Conceptually it is asking about how the wavelength Doppler shifts in an actively propagating wave . This picture might help:
upload_2014-12-28_12-25-0.png

Behind the plane, the wavelength would be shifted longer; conversely in front of the plane wavelengths would be "compressed." Replace plane with rupture for #46. Only waves parallel or anti-parallel to movement are shifted so A & B can be eliminated. The rupture is moving north so waves in front of it will be compressed/decreased. Thus the answer is C.
 
@ieatshrimp24 the question states it is not a static epicenter but moving rupture. For example it could be like a fracture along a fault line that starts at one point and moves north:
earthquake-plate-tectonic-iceland-story.jpg

Conceptually it is asking about how the wavelength Doppler shifts in an actively propagating wave . This picture might help:
View attachment 188047
Behind the plane, the wavelength would be shifted longer; conversely in front of the plane wavelengths would be "compressed." Replace plane with rupture for #46. Only waves parallel or anti-parallel to movement are shifted so A & B can be eliminated. The rupture is moving north so waves in front of it will be compressed/decreased. Thus the answer is C.

WOW This makes perfect sense. I didn't think about a moving epicenter. Once you said that it's not static, I automatically understood the question. Thanks @ahisma. Looks like I need to open my mind a little more for the MCAT. This was the passage that really tripped up my score.
 
#138: I'm wondering why "both P2 and EMS" isn't the correct answer. I understand that if you follow the chart in Figure 1, you would end up at "E", which is the only cell type that differentiates into gut. However, in Figure 2 we see that we need cell communication between P2 and EMS in order to optimize gut differentiation. Doesn't that mean that P2 also contains unique cell contents (i.e. diffusable morphogens, some sort of chemical signal, etc.) that are necessary for gut differentiation, since EMS alone is not sufficient for immediate gut differentiation (as shown in the previous question)?

Thanks :)
 
Hi,, I had three questions about the BS section I was hoping someone could help me out on. First for question 128, why is letter C not a viable answer. In the passage itself it states, "Although many individuals develop antibodies again H pylori antigens, these antibodies rarely eradicate the infection; evidently, this pathogen has developed effective ways to elude host defenses" Does this quote not support that the bacteria have a mechanism to be resistant against the immune system?

Also question 102, how do I know the placement of hydrogen. I understand how to do chiral centers, but how do I know the hydrogen is pointing away for both carbons?

Lastly, for question 105 how do you know that C or D are not viable answers? What makes diels alder only react in a certain way?

Thanks for the help
 
@steelersfan1243

I will try to give brief responses as I am walking out the door to dinner in a couple minutes.

128 - We are told the bacteria infects the stomach and no more. Any assumption of immune suppression would be outside of the scope of the passage. Therefore we cannot assume it infects immune cells and suppresses the immune response.

102 - SP3 hybridized carbons as drawn in that picture will have one substituent pointing to us and one away. The hydrogen atoms are not drawn (as is convention) and fill the last available position - away from us.

105 - We are told it is regiospecific - it forms as shown. Therefore the only possible difference can be stereochemistry, and the correct choice is the only enantomeric pair that reflects the stated regiospecificity.
 
  • Like
Reactions: 1 user
Hey guys, I am confused about 42 and 44 on AAMC 7.

I am having trouble with numbers 44 and 42.

For 44. I don't understand what the relationship is between resonance and elastic collisions. Wave nodes have the least amplitude so the answer cannot be C and destructive interference also decreases amplitude so it cannot be D. But i don't understand why A was chosen.

For 42, I don't understand why we would assume the quake propagates in all directions, does it have anything to do with longitudinal waves?

Is there anyway anyone could help me by explaining how one can use physics concepts to logically reason to the answer?


Much thanks.
 
@Silence in ER

42 - There isn't information provided to support that - though that is what occurs. I think this was more of a verbal style question where you had to eliminate the worst three answers. The other choices can be used to support the coincidence hypothesis, while that wouldn't. I didn't like that question.

44 - This question is also horrible. No logical reasoning.

Those weren't physics questions and I would be interested to hear if someone could reason an answer. I generally score well on most PS passages and I thought that 44 was near impossible to answer. I guess A is just the "best worst" answer - but I don't believe the passage or any knowledge we should have directly supports the answer.
 
  • Like
Reactions: 1 user
Yeah, I would love to find someone who could explain the physics o
@Silence in ER

42 - There isn't information provided to support that - though that is what occurs. I think this was more of a verbal style question where you had to eliminate the worst three answers. The other choices can be used to support the coincidence hypothesis, while that wouldn't. I didn't like that question.

44 - This question is also horrible. No logical reasoning.

Those weren't physics questions and I would be interested to hear if someone could reason an answer. I generally score well on most PS passages and I thought that 44 was near impossible to answer. I guess A is just the "best worst" answer - but I don't believe the passage or any knowledge we should have directly supports the answer.
Thanks Cawolf. I do hope if someone else has some more understanding of that question, I would love to understand it better.


One more thing, question 116 on AAMC 7,

I really don't understand why C couldn't work, after all, couldn't ( diurnal rhythms in physiological responses) be a confounding variable which affected their analysis?

Is it just a case of that it wasn't the best answer?
 
Can someone please explain verbal #81 to me on AAMC 7?

81: According to the passage, an American born in 1980 could expect to live 26.3 years longer than one born in 1900. Yet a white male of sixty could expect to live only 2.8 years longer in 1978 than in 1900. The difference in these expectancies can be explained as:

the answer is D

Thanks!
 
Yeah, I would love to find someone who could explain the physics o

Thanks Cawolf. I do hope if someone else has some more understanding of that question, I would love to understand it better.


One more thing, question 116 on AAMC 7,

I really don't understand why C couldn't work, after all, couldn't ( diurnal rhythms in physiological responses) be a confounding variable which affected their analysis?

Is it just a case of that it wasn't the best answer?

C was accounted for by controlling the time of day the results were gathered. If we assume that their physiological rhythm hasn't changed substantially, we can assume that the blood flow should be similar if we take the sample at a similar time. Therefore, the dependent variable, blood flow, is the best answer here.

For these type of questions, I think it's better to not even look back to the passage. You save time and don't need to even think too hard about it. Knowing that we are measuring blood flow differences, it would be silly and useless if it was a control.
 
  • Like
Reactions: 1 user
ok. thank you.
C was accounted for by controlling the time of day the results were gathered. If we assume that their physiological rhythm hasn't changed substantially, we can assume that the blood flow should be similar if we take the sample at a similar time. Therefore, the dependent variable, blood flow, is the best answer here.

For these type of questions, I think it's better to not even look back to the passage. You save time and don't need to even think too hard about it. Knowing that we are measuring blood flow differences, it would be silly and useless if it was a control.


ok. thank you.
 
For 144, so during 3-13 hours, the radioactive nucleotides were in the nuclei of the cells. But in other time periods, they were not? So was the nucleotide still incorporated by the cells but just not in the nucleus during these time periods?
 
For 144, so during 3-13 hours, the radioactive nucleotides were in the nuclei of the cells. But in other time periods, they were not? So was the nucleotide still incorporated by the cells but just not in the nucleus during these time periods?
The question is asking about incorporation of the nucleotides, which means that they must be actively taken up by the cell, not just be present. If they were just present in the cell, but not incorporated into some part of the cell, they'd be washed off with the rinse. Cells only take up nucleotides during the synthesis of RNA and DNA (as far as I'm aware), so that's the only time a loose nucleotide could be incorporated into the cell. The fact that they're found in the nucleus supports this, since that's where replication occurs. You know it's DNA synthesis not RNA synthesis because it's a deoxythymidine.

Does that help?
 
I ended up getting 44 right. Not by rigorous reasoning, but not a complete guess either. Something in "precarious equilibrium" is most likely to be disrupted by smaller changes than something stable. (ie. straw that broke the camel's back, etc) so I didn't eliminate A.

In order, I then eliminated D, C, and B. D because it didn't make physical sense at all -- given a destructive wave would weaken it. C because the passage talks about two types of waves in the earth, neither being standing. And B because nowhere in the passage mentioned "reinforcing resonant effects" and instead it basically said that the force dissipated away.

Hardly a good explanation, I know.


@Silence in ER

42 - There isn't information provided to support that - though that is what occurs. I think this was more of a verbal style question where you had to eliminate the worst three answers. The other choices can be used to support the coincidence hypothesis, while that wouldn't. I didn't like that question.

44 - This question is also horrible. No logical reasoning.

Those weren't physics questions and I would be interested to hear if someone could reason an answer. I generally score well on most PS passages and I thought that 44 was near impossible to answer. I guess A is just the "best worst" answer - but I don't believe the passage or any knowledge we should have directly supports the answer.
 
I still don't get #144. Wouldn't the cells of interest have to be in the synthesis (S) phase of interphase during the labeled dTTP treatment to include the label in their DNA? How, then do they reason that 3 hours later, that the same group of cells that are actively undergoing mitotic division, are in the S phase? Mitosis occurs AFTER replication, right? Am i missing something here? I feel really stupid
 
So apparently sympathetic activation makes you pee more? I always thought "rest and digest" of parasympathetic also applies to urination, but it seems like it's the inverse compared to digestion.

Activation of sympathetic = more pee (because higher blood pressure = high GFR) but less poo (inhibits peristalsis)

Activation of parasympathetic = less pee, more poo?


Is this the correct way of thinking about it?
 
So apparently sympathetic activation makes you pee more? I always thought "rest and digest" of parasympathetic also applies to urination, but it seems like it's the inverse compared to digestion.

Activation of sympathetic = more pee (because higher blood pressure = high GFR) but less poo (inhibits peristalsis)

Activation of parasympathetic = less pee, more poo?


Is this the correct way of thinking about it?

Your line of thinking is correct. However, sympathetic stimulation relaxes the dextrusor muscle of the urinary bladder, essentially holding back urination. So there's less of an urge to pee when sympathetic stimulation is activated. I'm not sure how a higher GFR is compensated for though.
 
However, sympathetic stimulation relaxes the dextrusor muscle of the urinary bladder, essentially holding back urination. So there's less of an urge to pee when sympathetic stimulation is activated. I'm not sure how a higher GFR is compensated for though.


Hmm.. my question was referring to the skin diving passage:

During the initial skin diving session, when her heart and breathing rates were increased, Sarah noticed that she produced more urine than usual. This was probably a result of:

A. increased blood pressure caused by her excitement or anxiety
B. reduced blood pressure caused by her excitement or anxiety
C. absorption of water from the ocean
D. inability to cool the skin through evaporative water loss.

I suppose the "noticed that she PRODUCED more urine than usual" doesn't mean "she NEEDED to pee more", but rather "she somehow noticed that her bladder was storing more urine than usual?"
 
nfg05- I picked answer choice C as well. I don't understand why C is wrong.
I did not think A was correct because as you stated above its implication is that the body is basically helpless against any bacterial infection in the stomach.

nfg05's post:
For item 130.

Which of the following statements explains most plausibly why host antibodies are ineffective against H. pylori?
A. antibody proteins may be denatured in the harsh environment of the stomach.
C. H. pylori infection may suppress the activity of the immune system.

Could someone explain why they knew C was wrong? There are some bacteria (i.e. the species responsible for TB and leprosy) that suppress immune system activity, and I wouldn't think the AAMC expects us to have detailed knowledge of whether or not H. pylori is one of those bacterial species. It is not impossible, however, for bacteria to display such a capability.

When looking at A, the implication is that the body is basically helpless against any bacterial infection in the stomach (provided the bacteria can survive in low pH environments). Also, many proteins make it through the stomach and aren't digested until the small intestine so I was unsure whether the antibody proteins would be denatured or not. So, are all proteins DENATURED (secondary and up structure broken) in the low pH stomach and then those not digested (primary structure broken) in the stomach are digested in the small intestine?


futuredoctor10 and nfg05 - I picked answer choice C as well, especially given that the passage states, "Although many individuals develop antibodies against H. pylori antigens, these antibodies rarely eradicate the infection; evidently, this pathogen has developed effective ways to elude host defenses." --> this bolded statement made me think C., "H. pylori infection may suppress the activity of the immune system." is correct.[/QUOTE]
 
Top